Answer:
The answer is "8(25+x)>500;x>37.50".
Step-by-step explanation:
Please find the complete question in the attached file.
[tex]\to 8(25+x)>500\\\\\to 200 +8x>500\\\\\to 8x> 300\\\\\to x> \frac{300}{8}\\\\\to x> 37.50[/tex]
That's why above given choice is correct.
Natalie bought a 3 - pack of erasers for $1.47. How much did each eraser cost?
Answer:
Each eraser costs $0.49
Step-by-step explanation:
If three erasers cost $1.47, and you have to find the value of one eraser, all you have to do is divide $1.47 by three.
1.47 ÷ 3 = 0.49
To prove this, you can multiple 0.49 by three.
0.49 × 3 = 1.47
Therefore, your answer will be $0.49 per eraser.
(Remember to add the money symbol because some hardcore teachers won't accept the answer without it, or will deduct points. The symbol looks like this: $)
Answer:
$0.49
Explanation:
1.47 divided by 3 is 0.49.
°ω°
:⊃
°∨°
What is 49÷34÷45=? Pleeeeeeeessse I need help
Answer:
The answer is 0.032026144
Step-by-step explanation:
Hoped This helped!
Answer:
0.032
Step-by-step explanation:
So we just go in order:
49 ÷ 34 = 1.44 (simplified)
1.44 ÷ 45 = 0.032 (simplified)
So 49 ÷ 34 ÷ 45 = 0.032
hope this helps:)
PLEASE ANSWER ASAP !!!!!!!!!!!!!!!!!!! WILL GET BRAINLEST IF CORRECT!!!!!!!!!!!!!!!!!!!!!!
Answer: x=40
Step-by-step explanation:
Answer:
X equals 40.
Step-by-step explanation:
since the other side is 120 degrees then the side opposed to it would also be 120. in case you're wondering what the other two sides are, they are both 60.
Plz guys can you guys help me
Answer:
answer B is correct of this question
Answer:I believe the answer is C my dude
Step-by-step explanation:
Can someone please help me
9514 1404 393
Answer:
Exponential Decay
Step-by-step explanation:
The base of the exponential term is less than 1, so the function represents decay.
Simplify the expression 4(n-3) + 2(-3+n)
Answer:
i think it is 2n +18
Step-by-step explanation:
4n - 12 + 6 +2n
so 4n -2n +12 +6 =
2n +18
Answer:
6n-6
Step-by-step explanation:
4(n-3) + 2(3+n)
1. Multiply 4 by n and 4 by 3 and Multiply 2 by 3 and 2 by n
4n-12 + 6+2n
2. Now you add your common numbers
6n -6
The weight of an object on the moon is 1/6 its weight on earth. If a bowling ball weighs 12 and 1/2 pounds on earth, how much does it weigh on the moon?
Determine the intercepts of the line.
v-intercept: (____,___)
y-intercept: (____,___)
manda made $270 for 18 hours of work. At the same rate, how many hours would she have to work to make $165?
find the height of the cylinder whose volume is 1.54 cubic m and radius of the base is 70 cm
need full solution or I'll report
find the height of the cylinder whose volume is 1.54 cubic m and radius of the base is 70 cm
Required diagram :
[tex]\setlength{\unitlength}{1mm}\begin{picture}(5,5)\thicklines\multiput(-0.5,-1)(26,0){2}{\line(0,1){40}}\multiput(12.5,-1)(0,3.2){13}{\line(0,1){1.6}}\multiput(12.5,-1)(0,40){2}{\multiput(0,0)(2,0){7}{\line(1,0){1}}}\multiput(0,0)(0,40){2}{\qbezier(1,0)(12,3)(24,0)\qbezier(1,0)(-2,-1)(1,-2)\qbezier(24,0)(27,-1)(24,-2)\qbezier(1,-2)(12,-5)(24,-2)}\multiput(18,2)(0,32){2}{\sf{ 70 \: cm}}\put(9,17.5){\sf{h}}\end{picture}[/tex]
Solution :For a cylinder having a radius r and a height h ;
Volume Of Cylinder :
π r² .Here , the radius of the cylinder is given as 70 cm and the volume is 1.54 m³ .
Converting ;
1 m³ = 1 × 10⁶ cm³ .> 1.54 × 10⁶ cm³ .> 1540000 cm ³.> π r² h = 1540000> 22/7 × 70 × 70 × h = 1540000> 22 × 10 × 70 h = 1540000> 15400 h = 1540000> h = 100 cm = 1 m.The height of the cylinder is 100cm.=1m.
Please help 15 points
Answer:
12 %
Step-by-step explanation:
3/25 = 0.12
0.12 x 100 = 12
Answer:
12%
Step-by-step explanation:
3/25=0.12
0.12 turnes into 12%
Which expression has a value of 13?
-12/6+ 7 - (-8)
-3 •2 - 19
4•5 -(-7)
5+11+(-3) - 6
Step-by-step explanation
a car is 180inches long. a truck is 75% longer then the car. how long is the truck?
Answer:
135 in.
Step-by-step explanation:
Emily has two bags of marbles. The first bag contains 4 red marbles and 6 blue.
The second bag contains 6 blue marbles, and 2 green marbles. Emily will
randomly select 1 marble from each bag.
What is the probability that Emily will select a blue marble from each bag?
Answer:
more in second bag
Step-by-step explanation:
because theres less numbers only 2 greens
The weight y of an object on Titan, one of Saturn's moons, is proportional to the weight
x of the object on Earth. An object that weighs 105 pounds on Earth would weigh 15
pounds on Titan. How much would a spacecraft that weighs 3500 kilograms on
Earth weigh on Titan?
Answer:
The spacecraft would weigh 500 kilograms on Titan.
Step-by-step explanation:
An object that weighs 105 pounds on Earth would weigh 15 pounds on Titan.
This means that the weighs on Titan are 105/15 = 7 times less than the weighs on Earth.
How much would a spacecraft that weighs 3500 kilograms on Earth weigh on Titan?
3500/7 = 500 kilograms
The spacecraft would weigh 500 kilograms on Titan.
(3x^2-5x+5)-(7x^2+4x-10)
Can anyone please explain this problem?
Dessie went to a Restaurant with some friends. The three of them split a large pizza. Dessie also ordered breadsticks for $5. Dessie spent a total of $11. How mich was the large pizza? (pls help!)
Answer: I think its $6
Step-by-step explanation: Because, if she spent a total of $11, and we know that she spent $5, you would subtract the 11 and the 5 to get 6. Hope this helps!
Name two ways that saving and investing are different
What are the solutions to x2 – 10x = 39?
Answer: x=−3 or x=13
Step-by-step explanation:
Step 1: Subtract 39 from both sides.
x2−10x−39=39−39
x2−10x−39=0
Step 2: Factor left side of equation.
(x+3)(x−13)=0
Step 3: Set factors equal to 0.
x+3=0 or x−13=0
x=−3 or x=13
Which set of values for x makes the inequality 12 - x < 7 true?
A. 3,4,5
B. 5
C. 5,6,7
D. 6,7
Answer:
D. 6, 7
Step-by-step explanation:
12 - x < 7
-x < -5
(whenever you multiply or divide an inequality by a negative you must switch the inequality symbol)
x > 5
Mateo is 64 inches tall. He casts a shadow that is 50 inches long. His shadow extends to the end of the tree's shadow when he stands 180 inches from the tree. What is the height of the tree?
Answer:244 inches
Step-by-step explanation:
Answer:
230
Step-by-step explanation:
64/50 because h/s
64 x
50 180
180 x 64 = 11520
11520 ÷ 50 = 230
(I'm sorry if its wrong but um 8/10 its right
Liz has two gardens and wants to build a fence around each. She wants to buy all the fencing at one time. The diagram below shows her two
gardens.
Vegetable Garden
2x feet
Fruit Garden
x + 4 feet
3x + 4 feet
Sx feet
Which of the following correctly shows the total amount of fencing, in feet, Liz will need to buy for both gardens ?
Total: 29 x
O Total: 11 x + 4
Total: 22 x + 16
Total: 11 x + 8
Previous
Answer:
C 22x + 16!
Step-by-step explanation:
PLEASE HELP i have to turn this in in an hour PLEASE DONT GUESS
Answer:
x = 29°
Step-by-step explanation:
the Δ on the right has 3 angles whose measures are 30°, 74°, and 76° (you get 76° because it forms a linear pair with an angle that is 104°; you get 74° by adding 30 and 76 and subtracting the sum from 180°.
the Δ on the left has 3 angles whose measures are 45°, 29°, and 106° (you get 106° because it forms a linear pair with an angle that is 74°; you get 29° by adding 45 and 106 and subtracting the sum from 180°.
∡x is a vertical angle with the angle that measures 29° and vertical angles are congruent
Answer:
hopw this helps!!!!!!! so this means that x = 29 :) hoep t∡x is a vertical angle with the angle that measures 29° and vertical angles are congruent
Step-by-step explanation:
I hate these plz help again
Answer and Step-by-step explanation:
We can eliminate answer choices 2 and 4, since the graphs are shading below the line, which means y is less than or less than or equal to the values.
The answer is the first answer choice.
This is because this inequality matches up with the graph.
Everything is shaded below, and y is less than, so it matches up.
3 is the y-intercept.
The slopes match up as well.
#teamtrees #WAP (Water And Plant)
Help me out! Answer? Anyone??
Answer: C and D.
Step-by-step explanation: The pictures show the work. So sorry about the delay -- my computer wasn't letting me type formulas, so I hand-wrote. If you have questions, let me know and I'd be happy to clarify, and if the pictures are sideways, use the rotate button at the top right.
If a video is getting 5,000 views every 10 minutes, how long will it take to reach 60,000 views?
Answer:
60,000÷5000=12
12×10=120 minutes.
sam writes on a white board the positive integers from 1 to 6 inclusive once each. she then writes p additional fives and q sevens on the board. The mean of all the numbers on the board is then 5.3. What is the smallest possible value of q?
Answer:7
Step-by-step explanation:
This question is on the ukmt junior maths challenge which I'm doing now, out of the options given I got 7, because 2+4+6+5+(5×20)+(7×7)= 166, (then dividing that by the number of numbers to get the mean) so 166÷31= 5.354.....
Also the 3 has a recurring symbol above it
The 7 is times seven so uh yeah<3
The smallest possible value of 'q' is 7 and this can be determined by using the hit and trial method and mean formula.
Given :
Sam writes on a whiteboard the positive integers from 1 to 6 inclusive once each. she then writes p additional fives and q sevens on the board.The mean of all the numbers on the board is then 5.3.The following steps can be used in order to determine the smallest possible value of q:
Step 1 - The formula of the mean can be used in order to determine the smallest possible value of q.
Step 2 - The mean of all the numbers is given by:
[tex]\rm \dfrac{2+4+6+5+5p+7q}{6+p+q}=5.3[/tex]
[tex]\rm \dfrac{21 + 5p + 7 q}{6+p+q} = 5.3[/tex]
Step 3 - So, by the hit and trial method, the value of q can be determined.
Therefore, the value of 'q' is 7.
For more information, refer to the link given below:
https://brainly.com/question/25277954
How to find the domain of a composite function
Answer:
The domain of a composite function f(g(x)) f ( g ( x ) ) is the set of those inputs x in the domain of g for which g(x) is in the domain of f .
ill give brainliest
Triangle Sum Theorem CLASSWORK
Step-by-step explanation:
Sum of angles in a triangle = 180°.
Question 3:
We have 90° + 61° + (14x + 1)° = 180°.
=> 14x° + 152° = 180°
=> 14x° = 28°
=> x = 2.
Question 4:
We have 70° + 55° + (4x + 3)° = 180°.
=> 4x° + 128° = 180°
=> 4x° = 52°
=> x = 13.
Answer:
[tex](14x + 1) + 90 + 61 = 180 \\ 14x + 152 = 180 \\ 14x = 28 \\ \boxed{x = 2}[/tex]
2 is the right answer.[tex](4x + 3) + 55 + 70 = 180 \\ 4x + 128 = 180 \\ 4x = 52 \\ \boxed{x = 13}[/tex]
13 is the right answer.graph y-2=-3(x-4). please help me